What are the domain and range of f(x)=2(x−8)2−10?



Drag the answers into the boxes

Answers

Answer 1

The domain and range of f(x) = 2(x-8)² - 10 are Domain: (-∞, ∞) ,Range: [-10, ∞)

The given function, f(x) = 2(x-8)² - 10, is a quadratic function in the form of f(x) = a(x-h)² + k. In this case, a = 2, h = 8, and k = -10. Since the coefficient of the squared term (a) is positive, the parabola opens upwards.

The domain of a quadratic function is always all real numbers, so the domain is (-∞, ∞).

For the range, we need to find the minimum value of the function. Since the parabola opens upwards, the vertex of the parabola represents the minimum point. The vertex is located at (h, k), which in this case is (8, -10). Thus, the range of the function is all real numbers greater than or equal to the y-coordinate of the vertex, which is [-10, ∞).

To know more about quadratic function click on below link:

https://brainly.com/question/30929439#

#SPJ11


Related Questions

Marcia drew a plan for a rectangular piece of material that she will use for a blanket. Three of the vertices are (-1.3, -3.5), (-1.3, 1.4), and (3.9,1.4) . What are the coordinates of the fourth​ vertex?

Answers

The coordinates of the fourth vertex is (3.9, 6.3,0).To find the coordinates of the fourth vertex, we can use the fact that opposite sides of a rectangle are parallel and equal in length.

We can find the length and slope of one side of the rectangle and then use that information to find the coordinates of the fourth vertex.

Let's start by finding the length and slope of the side connecting (-1.3, -3.5) and (-1.3, 1.4). The length of this side is the difference between the y-coordinates, which is:

1.4 - (-3.5) = 4.9

Since this side is vertical, its slope is undefined.

Next, let's find the length and slope of the side connecting (-1.3, 1.4) and (3.9, 1.4). The length of this side is the difference between the x-coordinates, which is:

3.9 - (-1.3) = 5.2

Since this side is horizontal, its slope is 0.

Since opposite sides of a rectangle are equal in length, the length of the side connecting (-1.3, 1.4) and (3.9, 1.4) must also be 4.9. We can use this length to find the y-coordinate of the fourth vertex, which is:

1.4 + 4.9 = 6.3

Now we know that the fourth vertex has coordinates (x, 6.3). To find the x-coordinate, we can use the length of the vertical side connecting (-1.3, -3.5) and (-1.3, 1.4), which is also 5.2. The x-coordinate of the fourth vertex is:

-1.3 + 5.2 = 3.9

Therefore, the coordinates of the fourth vertex are (3.9, 6.3,0).

Learn more about Vertex:

https://brainly.com/question/30945046

#SPJ1

Q=1/6p^2


p= 13. 6 correct to 3 significant figures.


By considering bounds, work out the value of q to a suitable degree of accuracy.


Give a reason for your answer.


+

Answers

The value of Q, taking into account the significant figures is 30.8.

To work out the value of Q given the value of p, we can substitute the value of p into the equation Q = (1/6) × p².

Given p = 13.6, we can calculate Q as follows:

Q = (1/6) × (13.6)²

Q = (1/6) × 184.96

Q = 30.826666...

Now, let's consider the significant figures of the given value of p, which is 13.6 (3 significant figures).

Since the value of p has 3 significant figures, we should round our final answer for Q to 3 significant figures as well.

Considering the value of Q to a suitable degree of accuracy, we can round our answer to three significant figures, which gives us:

Q = 30.8

Therefore, the value of Q, taking into account the significant figures, is  30.8.

To learn more on Number system click:

https://brainly.com/question/22046046

#SPJ12

Please help I need the code asap

Answers

The values based on the exponent will be:

0.024

5670

41952

0.005

73

0.34

900

6

How to calculate the values

The exponent is the number of times that a number is multiplied by itself. It should be noted that the power is an expression which shows the multiplication for the same number. For example, in 6⁴ , 4 is the exponent and 6⁴ is called 6 raise to the power of 4.

1) 2.4 × 10^-2 = 0.024

2) 5.67 x 10^3 = 5670

3) 4.1952 x 10^4 = 41952

4) 5 × 10^-3 = 0.005

5) 7.3 × 10^1 = 73

6) 3.4 × 10-1 = 0.34

7) 9 × 10^2 = 900

8) 6 × 10*0 = 6

Learn more about exponents on

https://brainly.com/question/13669161

#SPJ1

Mirrors kitchen sink holds up to 108.460 L of water runs amount to the nearest liter

Answers

The statement mentions that the kitchen sink has a capacity of 108.460 L of water and it is important to round up the amount to the nearest liter. When we round up the capacity of the sink, it comes out to be 108 liters. This means that the sink can hold up to 108 liters of water at maximum capacity.


It is important to have an idea of the sink’s capacity in terms of liters because it helps in managing the amount of water used while washing dishes or other household items. It is also beneficial to know the capacity of the sink while filling it with water for cleaning purposes, as it prevents the sink from overflowing.


Overall, the capacity of a sink is an important factor to consider while designing a kitchen or bathroom as it ensures proper functionality and prevents any damage to the surrounding areas due to overflowing water. So, it is always advisable to check the capacity of a sink before installing it in a household.

To know more about capacity click here

brainly.com/question/29033783

#SPJ11

Complete Question  :  Mirrors kitchen sink holds up to 108.460 L of water. Round this amount to the nearest liter.

A gift bag is shaped like a rectangular prism and has a volume of 1152 cubic inches.

Answers

The volume of the gift bag is given as 1152 cubic inches.

Since it is shaped like a rectangular prism, we can write the formula for its volume as V = l × w × h, where l, w, and h are the length, width, and height of the rectangular prism, respectively.

To determine the dimensions of the gift bag, we need more information such as the ratio of its length, width, and height or any one of its dimensions. If we assume one of the dimensions, say, the length is L inches, then we can write the volume as V = L × w × h. Solving for w × h, we get w × h = V/L = 1152/L.

We can then use this equation along with the fact that the gift bag is a rectangular prism to find the other dimensions. For example, if the width is W inches, then we have h = 1152/(L × W) and the volume can be expressed as V = L × W × 1152/(L × W) = 1152.

Similarly, if the height is H inches, then we have w = 1152/(L × H) and the volume can be expressed as V = L × 1152/(L × H) × H = 1152.

For more questions like Volume click the link below:

https://brainly.com/question/1578538

#SPJ11

The am between two number exceeds their gm by 2 and the gm exceed the hm by 1.8 find the number

Answers

The unknown number x = (6.1 ± sqrt(6.

How to find the unknown number

Let's call the two numbers x and y.

We are given:

AM (Arithmetic Mean) between x and y exceeds their GM (Geometric Mean) by 2:

(x + y)/2 - sqrt(xy) = 2

GM between x and y exceeds their HM (Harmonic Mean) by 1.8:

sqrt(xy) - 2xy/(x+y) = 1.8

We can solve for one variable in terms of the other from the second equation and substitute into the first equation to solve for the remaining variable. Let's solve for y in terms of x from the second equation:

sqrt(xy) - 2xy/(x+y) = 1.8

sqrt(xy)(x+y) - 2xy = 1.8(x+y)

sqrt(xy)x + sqrt(xy)y - 2xy = 1.8x + 1.8y

sqrt(xy)(x+y-1.8) = 0.2x + 0.8y

x+y-1.8 = (0.2/0.8)sqrt(xy)(x+y-1.8)

x+y-1.8 = 0.25sqrt(xy)(x+y-1.8)

4x + 4y - 7.2 = sqrt(xy)(x+y)

Now we can substitute this expression for sqrt(xy)(x+y) into the first equation and solve for x:

(x + y)/2 - sqrt(xy) = 2

(x + y)/2 - (4x + 4y - 7.2)/4 = 2

2(x + y) - (4x + 4y - 7.2) = 8

12.2 = 2x + 2y

6.1 = x + y

Now we can substitute x + y = 6.1 into the expression we derived for sqrt(xy)(x+y) to solve for sqrt(xy):

4x + 4y - 7.2 = sqrt(xy)(x+y)

4x + 4y - 7.2 = sqrt(xy)(6.1)

sqrt(xy) = (4x + 4y - 7.2)/6.1

Finally, we can substitute both x + y = 6.1 and sqrt(xy) = (4x + 4y - 7.2)/6.1 into the equation sqrt(xy) - 2xy/(x+y) = 1.8 and solve for y:

sqrt(xy) - 2xy/(x+y) = 1.8

(4x + 4y - 7.2)/6.1 - 2xy/6.1 = 1.8

4x + 4y - 7.2 - 12.2xy = 11.38

4x + 4y - 11.38 = 12.2xy

4x + 4(6.1 - x) - 11.38 = 12.2xy (substituting x + y = 6.1)

xy = 4.08

Now we know that xy = 4.08, and we can use this to solve for x and y:

y = 6.1 - x

xy = 4.08

x(6.1 - x) = 4.08

6.1x - x^2 = 4.08

x^2 - 6.1x + 4.08 = 0

We can solve for x using the quadratic formula:

x = (6.1 ± sqrt(6.

Learn more about Arithmetic Mean at https://brainly.com/question/17139085

#SPJ1

Help with problem in photo!

Answers

Check the picture below.

[tex]4+10x=\cfrac{(9x+20)+10x}{2}\implies 8+20x=19x+20\implies x=12 \\\\[-0.35em] ~\dotfill\\\\ 4+10x\implies 4+10(12)\implies \stackrel{ \measuredangle DEC }{124^o}[/tex]

Jaoan reads that themass of an average elephant's brain is 3 4/10 kilograms greater than an average man's brain. How many kilograms is an average elephant's brain?

Answers

Answer:

Step-by-step explanation:

Let's call the mass of an average man's brain "m."

According to the problem, the mass of an average elephant's brain is 3 4/10 kilograms greater than an average man's brain. We can write this as:

mass of elephant's brain = m + 3 4/10

To find out how many kilograms an average elephant's brain weighs, we need to know the value of "m." However, this information is not given in the problem.

Therefore, we cannot determine the exact mass of an average elephant's brain.

Frank wants to paint his room in the
school colors of maroon and white. The floor and ceiling will be white, and all the walls will be maroon. The door will also be white. If one gallon of paint covers 400 sq ft, how many gallons of each color will he need?

A. 1 gallon white,1 gallon maroon
B. 1 gallon white,2 gallons maroon
C. 2 gallon white,2 gallons maroon
D. 2 gallon white,3 gallons maroon ​

Answers

To determine how many gallons of white and maroon paint Frank will need, we need to calculate the total square footage for each color. Here's a step-by-step explanation:

1. Determine the square footage of the floor and ceiling that will be painted white. Since they are the same size, we can calculate the area of one and multiply it by 2.
2. Determine the square footage of all the walls that will be painted maroon. Calculate the area of each wall and sum them up.
3. Determine the square footage of the door that will be painted white. Subtract this value from the total maroon wall area.
4. Divide the total square footage of the white and maroon surfaces by 400 sq ft (coverage of one gallon) to find out how many gallons are needed for each color.

After calculating the areas and the number of gallons needed, compare the results with the given options (A, B, C, or D). Keep in mind that we don't have the specific dimensions for Frank's room, but following these steps will help you solve the problem once you have the necessary measurements.

Learn more about paint at https://brainly.com/question/27111142

#SPJ11

the length of a rectangle is 4 cm less than the twice of the width. if the perimeter is 178 cm what is the width of the rectangle?​

Answers

Answer: 31

Step-by-step explanation:

We know that the perimeter is equal to 2W + 2L.

       178 cm = 2W + 2L

We also know that the length is 4 cm less than twice the width.

       L = 2W - 4

We will create a system of equations. Then we will solve with substitution.

       178 cm = 2W + 2L

       L = 2W - 4

       178 cm = 2W + 2L

       178 cm = 2W + 2(2W - 4 cm)

       178 cm = 2W + 4W - 8 cm

       178 cm = 6W - 8 cm

       186 cm = 6W

               W = 31

Answer:

31 cm

Step-by-step explanation:

First, we choose two variables.

Let W = width.

Let L = length.

The perimeter of a rectangle is:

perimeter = 2(L + W)

Now we translate the statements of the problem into two equations.

"the perimeter is 178 cm"

2(L + W) = 178

Divide both sides by 2:

L + W = 89

"the length of a rectangle is 4 cm less than the twice of the width"

L = 2W - 4

We have a system of equations:

L + W = 89

L = 2W - 4

Rewrite the first equation:

L + W = 89

Since the second equation is already solved for L, we can easily use the substitution method.

Substitute 2W - 4 for L in the equation above.

2W - 4 + W = 89

Combine like terms on the left side.

3W - 4 = 89

Add 4 to both sides.

3W = 93

Divide both sides by 3.

W = 31

Answer: The width is 31 cm

Check:

The length is 4 cm less than twice the width. 2 × 31 cm - 4 cm = 58 cm

The length is 58 cm, and the width is 31 cm. Calculate the perimeter.

P = 2(L + W) = 2(58 cm + 31 cm) = 2(89 cm) = 178 cm

The perimeter is 178 cm as the problem states, so the answer, width = 31 cm, is correct.

Use Lagrange multipliers to find the indicated extrema, assuming that x, y, and z are positive.
Maximize: f(x, y, 2) = xyz
Constraint: × + y + z - 6 = 0
f = _____

Answers

To use Lagrange multipliers, we need to set up the Lagrangian function, Therefore, the maximum value of f(x,y,2) = xyz subject to the constraint x+y+z-6=0 is f(2,2,2) = 8.

L(x,y,z,λ) = xyz + λ(x+y+z-6)
Then we need to find the critical points of L by setting its partial derivatives equal to zero:
∂L/∂x = yz + λ = 0
∂L/∂y = xz + λ = 0
∂L/∂z = xy + λ = 0
∂L/∂λ = x + y + z - 6 = 0
Solving this system of equations, we get:
x = y = z = 2
λ = -4
This critical point satisfies the constraint, since 2+2+2-6 = 0. To check whether it is a maximum or minimum, we need to use the second partial derivative test:
∂²L/∂x² = 0, ∂²L/∂y² = 0, ∂²L/∂z² = 0
∂²L/∂x∂y = z, ∂²L/∂x∂z = y, ∂²L/∂y∂z = x

The Hessian matrix is:
| 0  z  y |
| z  0  x |
| y  x  0 |
At the critical point (2,2,2), the Hessian matrix is:
| 0  2  2 |
| 2  0  2 |
| 2  2  0 |
The eigenvalues of this matrix are -4, -4, and 8. Since the eigenvalues are not all positive or all negative, we cannot conclude whether the critical point is a maximum or minimum.
Therefore, the maximum value of f(x,y,2) = xyz subject to the constraint x+y+z-6=0 is f(2,2,2) = 8.

Visit here to know more about Lagrangian function:

brainly.com/question/31367466

#SPJ11

can yall help me with this and this is due today!

Answers

a) The experimental probability of rolling an even number is given as follows: 12/25.

b) The theoretical probability of rolling an even number is given as follows: 1/2.

c) With a large number of trials, there might be a difference between the experimental and the theoretical probabilities, but the difference should be small.

How to calculate a probability?

A probability is calculated as the division of the desired number of outcomes by the total number of outcomes in the context of a problem/experiment.

The number of trials in which an even number is rolled is given as follows:

88 + 69 + 83 = 240.

Hence the experimental probability is given as follows:

240/500 = 12/25.

For each roll, 3 out of 6 numbers are even, hence the theoretical probability is given as follows:

p = 3/6

p = 1/2.

More can be learned about probability at https://brainly.com/question/24756209

#SPJ1

Construct a 95% confidence interval for the true average monthly salary earned by all employees of People Plus Pty, if it was found that the average monthly salary earned by a sample of 19 employees of the company was R18 500, with a standard deviation of R1 750. Interpret your answer

Answers

We can be 95% confident that the true average monthly salary earned by all employees of People Plus Pty is between R16 234.49 and R20 765.51. This means if created for several samples of the same size taken from the population, would contain the actual population mean.

To construct a 95% confidence interval for the true average monthly salary earned by all employees of People Plus Pty, we can use the following formula:

CI = x ± t(α/2, n-1) * (s/√n)

where:

x = sample mean = R18 500

s = sample standard deviation = R1 750

n = sample size = 19

t(α/2, n-1) = t-score at α/2 and n-1 degrees of freedom

Using a t-table or calculator with 18 degrees of freedom (n-1), we can find the t-score at α/2 = 0.025 to be 2.101.

Plugging in the values, we get:

CI = 18500 ± 2.101 * (1750/√19)

= (16234.49, 20765.51)

Therefore, we can be 95% confident that the true average monthly salary earned by all employees of People Plus Pty is between R16 234.49 and R20 765.51.

This means that if we were to take multiple samples of the same size from the population and construct 95% confidence intervals for each sample mean, about 95% of those intervals would contain the true population mean.

Learn more about confidence interval at https://brainly.com/question/18915814

#SPJ11

will mark brainlist for anyone who do step by step correctly and make sure it's not no other answers that is not on the answer choice.
An image of a rhombus is shown.
What is the area of the rhombus?

224 cm2

120 cm2

112 cm2

60 cm2

Answers

Answer:224cm^2

Step-by-step explanation:

Formula for the area of a parallelogram is base x height (b x h) so we do 14x16 which is 224.

calculate div(f) and curl(f). f = 5ey, 2 sin(x), 9 cos(x)

Answers

Div(f) = 2cos(x) + [tex]5e^y[/tex], and curl(f) = < 0, 9sin(x), 5e^y >.

To calculate div(f) and curl(f), we need to express f as a vector field:

f = < 2 sin(x), [tex]5e^y[/tex][tex]5e^y[/tex], 9 cos(x) >

Then, we can use the formulas for divergence and curl:

div(f) = ∂f₁/∂x + ∂f₂/∂y + ∂f₃/∂z

curl(f) = < ∂f₃/∂y - ∂f₂/∂z, ∂f₁/∂z - ∂f₃/∂x, ∂f₂/∂x - ∂f₁/∂y >

Let's compute these step by step:

div(f) = ∂f₁/∂x + ∂f₂/∂y + ∂f₃/∂z

= 2cos(x) + [tex]5e^y[/tex] + 0

= 2cos(x) + [tex]5e^y[/tex]

curl(f) = < ∂f₃/∂y - ∂f₂/∂z, ∂f₁/∂z - ∂f₃/∂x, ∂f₂/∂x - ∂f₁/∂y >

= < 0 - 0, 0 - (-9sin(x)), 5e^y - 0 >

= < 0, 9sin(x), 5e^y >

Therefore, div(f) = [tex]2cos(x) + 5e^y[/tex], and curl(f) = [tex]< 0, 9sin(x), 5e^y > .[/tex]

Learn more about Div(f)

https://brainly.com/question/29997133

#SPJ4

Chris bought 5 tacos and 2 burritos for $13. 25.


Brett bought 3 tacos and 2 burritos for $10. 75.


The price of one taco is $


The price of one burrito is $

Answers

Answer:

Let's start by assigning some variables to the unknowns:

Let's call the price of one taco "t".

Let's call the price of one burrito "b".

With these variables, we can write two equations based on the information given in the problem:

5t + 2b = 13.25 (equation 1)

3t + 2b = 10.75 (equation 2)

We now have two equations and two variables. We can use algebra to solve for t and b. One way to do this is to eliminate b by subtracting equation 2 from equation 1:

(5t + 2b) - (3t + 2b) = 13.25 - 10.75

Simplifying this equation, we get:

2t = 2.5

Dividing both sides by 2, we get:

t = 1.25

So the price of one taco is $1.25.

Now that we know the price of one taco, we can substitute this value into one of the equations to solve for b. Let's use equation 1:

5t + 2b = 13.25

Substituting t = 1.25, we get:

5(1.25) + 2b = 13.25

Simplifying this equation, we get:

6.25 + 2b = 13.25

Subtracting 6.25 from both sides, we get:

2b = 7

Dividing both sides by 2, we get:

b = 3.5

So the price of one burrito is $3.5.

Therefore, the price of one taco is $1.25 and the price of one burrito is $3.5.

To know more about variables refer here

https://brainly.com/question/17344045#

#SPJ11

Which of the fraction, decimai, percent equivalencies are correct? Select THREE correct answers

Answers

The fractions, decimals, and percent equivalencies that are correct are:

b. 3/8 = 0.375 = 37.5%

c. 24% = 0.24 = 6/25

e.  24/30 = 80% = 0.8

What are fractions, decimals, and percentages?

A fraction is a part of a whole.

The number is represented mathematically as a quotient, where the numerator and denominator are split.

In a simple fraction both the numerator and denominator are integers.

In a complex fraction, a fraction appears in the numerator or denominator.

In a proper fraction, the numerator is less than the denominator.

A decimal is a fraction that contains a decimal point.

A percentage is a value out of 100 parts.

Learn more  about fractions and decimals at: https://brainly.com/question/29404890

#SPJ1

Doors&Windows, Inc. Makes, you guessed it, doors and windows. Each day, Doors&Windows orders 6,150 linear feet of framing material and has a few staff member who are able to work a collective 600 hours. For each window, Doors&Windows profits $40 requiring 6 labor hours and 30 linear feet of framing material. For each table, Doors&Windows profits $90 requiring 10 labor hours and 120 linear feet of framing material. Due to the availability of glass, they can only produce a maximum of 70 windows. When formulating this problem as a linear programming model, what are the decision variables?

Answers

Decision variables in Doors&Windows linear programming model.

How to formulate linear programming?

The decision variables in the linear programming model for this problem are the number of windows (W) and the number of tables (T) that Doors&Windows should produce to maximize their profit while meeting the constraints. The objective function to maximize would be the total profit, which can be expressed as 40W + 90T. The constraints include the available linear feet of framing material (6150) and labor hours (600), which can be written as 30W + 120T ≤ 6150 and 6W + 10T ≤ 600, respectively. Additionally, the maximum number of windows that can be produced (W ≤ 70) due to glass availability is also a constraint. These decision variables and constraints can be used to create a linear programming model to optimize Doors&Windows' production and profit.

Learn more about linear programming model

brainly.com/question/31952194

#SPJ11

A particle moves on a coordinate line with acceleration a = d^2s/dt^2 = 15 sqrt(t) - (3/sqrt(t)), subject to the conditions that ds/dt = 4 and s = 0 when t = 1. Find a. the velocity y = ds/dt in terms of t. b. the position s in terms of t.

Answers

a.The velocity function is: v = ds/dt = (2/5)t^(5/2) - 6t^(1/2) + 16.

b. The position function is: s = (4/35)t^(7/2) - 8t^(3/2) + 16t - 12.

a. To find the velocity, we need to integrate the acceleration function. We get:

v = ds/dt = ∫a dt = ∫(15√t - 3/t^(1/2)) dt

Integrating the first term, we get (2/5)t^(5/2), and integrating the second term, we get -6t^(1/2) + C. Thus, the velocity function is:

v = ds/dt = (2/5)t^(5/2) - 6t^(1/2) + C

We can find the constant C using the initial condition that ds/dt = 4 when t = 1. Substituting these values into the equation, we get:

4 = (2/5)(1)^(5/2) - 6(1)^(1/2) + C

C = 4 + 12 = 16

Therefore, the velocity function is:

v = ds/dt = (2/5)t^(5/2) - 6t^(1/2) + 16

b. To find the position function, we need to integrate the velocity function. We get:

s = ∫v dt = ∫((2/5)t^(5/2) - 6t^(1/2) + 16) dt

Integrating the first term, we get (4/35)t^(7/2), integrating the second term, we get -8t^(3/2), and integrating the third term, we get 16t. Thus, the position function is:

s = ∫v dt = (4/35)t^(7/2) - 8t^(3/2) + 16t + C2

We can find the constant C2 using the initial condition that s = 0 when t = 1. Substituting these values into the equation, we get:

0 = (4/35)(1)^(7/2) - 8(1)^(3/2) + 16(1) + C2

C2 = -12

Therefore, the position function is:

s = (4/35)t^(7/2) - 8t^(3/2) + 16t - 12

To learn more about velocity function visit:https://brainly.com/question/28939258

#SPJ11

In ΔPQR, the measure of ∠R=90°, the measure of ∠P=26°, and PQ = 8. 5 feet. Find the length of QR to the nearest tenth of a foot

Answers

In ΔPQR, the length of QR is approximately 4.1 feet to the nearest tenth of a foot.

In ΔPQR, given that ∠R=90°, ∠P=26°, and PQ=8.5 feet, you want to find the length of QR to the nearest tenth of a foot.

Step 1: Since ∠R is a right angle (90°), we can use trigonometric ratios to find QR. First, let's find ∠Q. We know that the sum of angles in a triangle is 180°, so ∠Q = 180° - (∠P + ∠R) = 180° - (26° + 90°) = 64°.

Step 2: Now that we have all the angles, we can use the sine formula to find QR. We'll use the sine of ∠P (26°) and the given side PQ (8.5 feet) as our reference. The sine formula is:

QR = (PQ * sin(∠P)) / sin(∠Q)

Step 3: Plug in the known values:

QR = (8.5 * sin(26°)) / sin(64°)

Step 4: Calculate the sine values and the division:

QR = (8.5 * 0.4384) / 0.8988 ≈ 4.1326

Step 5: Round the answer to the nearest tenth of a foot:

QR ≈ 4.1 feet

In ΔPQR, the length of QR is approximately 4.1 feet to the nearest tenth of a foot.

Learn more about Triangles:  https://brainly.com/question/2773823

#SPJ11

B. It will be a curve.

C. It will be a line.

D. There is no way to tell.

Answers

The given equation on a graph can be described best as C. It will be a line.

Why would this be a line ?

The given equation is a linear equation because it has the form y = mx + b, where m is the slope and b is the y-intercept. In this case, the equation can be rewritten as:

y = 3x + 15 - 64

y = 3x - 49

Since it's a linear equation, it will represent a straight line on the graph that can be found by using the equation and some values of x, to find values of y and then plotting them.

In conclusion, the best answer is C. It will be a line.

Find out more on lines at https://brainly.com/question/28732353

#SPJ1

The full question is:

An equation is given to be 3x + 15 = 64

What will this be on a graph ?

A. It will be irregular

B. It will be a curve.

C. It will be a line.

D. There is no way to tell.

The graph of the function p(x) is represented below. On the same set of axes, sketch the function p(x + 2).

Answers

A graph of the function p(x) and the function p(x + 2) is shown below.

How to determine the factored form of a quadratic equation?

In Mathematics, the vertex form of a quadratic function is represented by the following mathematical equation:

f(x) = a(x - h)² + k

Where:

h and k represents the vertex of the graph.a represents the leading coefficient.

Based on the information provided above, we can determine the value of a as follows:

f(x) = a(x - h)² + k

0 = a(-3 - 0)² + 4

0 = 9a + 4

a = -4/9

Therefore, the required quadratic function is given by:

f(x) = a(x - h)² + k

p(x) = y = -4/9(x - 0)² + 4

p(x + 2) = -4/9(x + 2)² + 4

Read more on vertex here: brainly.com/question/14946018

#SPJ1

Missing information:

The question is incomplete and the complete question is shown in the attached picture.

Explain me that excersice step by step please3x((2x3)^-1x1/2^3)^-1x(3x2^2)^-2

Answers

Answer:

Begin by simplifying the phrases within the brackets, beginning with the innermost brackets:

(2 x 3)^-1 = 1/6 (because 2 x 3 = 6, and the negative exponent flips the fraction)

1/2^3 = 1/8 (because 2^3 = 8)

So, (2x3)^-1x1/2^3 = 1/6 x 1/8 = 1/48

Next, simplify the expression outside the parentheses:

(3x2^2)^-2 = 1/(3x2^2)^2 = 1/(3^2 x 2^4) = 1/36 x 1/16 = 1/576

Now, substitute the simplified terms back into the original expression and simplify:

3x(1/48)x(1/576) = 1/768

So the final answer is 1/768.

1)You have a monthly income of $2,800 and you are looking for an apartment. What is the maximum


amount you should spend on rent?



2)You have a monthly income of $1,900 and you are looking for an apartment. What is the maximum


amount you should spend on rent?


3)An apartment you like rents for $820. What must your monthly income be to afford this apartment?


4)An apartment you like rents for $900. What must your monthly income be to afford this apartment?


5)An apartment rents for $665/month. To start renting, you need the first and last month's rent, and a


$650 security deposit.

Answers

Using Rule of thumb,

The maximum amount you should spend on rent for an income of $2,800 is $840 (30% of $2,800).The maximum amount you should spend on rent for an income of $1,900 is $570 (30% of $1,900).To afford an $820/month apartment, your monthly income must be at least $2,733.33 (30% of $2,733.33).To afford a $900/month apartment, your monthly income must be at least $3,000 (30% of $3,000).To start renting the apartment, you would need $1,980 for first and last month's rent plus a $650 security deposit.

1) A common rule of thumb is to spend no more than 30% of your income on rent. Therefore, the maximum amount you should spend on rent is:

$2,800 x 0.3 = $840

2) Using the same rule of thumb, the maximum amount you should spend on rent is:

$1,900 x 0.3 = $570

3) To afford an $820/month apartment, you should aim to spend no more than 30% of your income on rent:

$820 = 0.3x

x = $2,733.33

So your monthly income must be at least $2,733.33 to afford the apartment.

4) Following the same reasoning, your monthly income must be at least:

$900 = 0.3x

x = $3,000

5) To start renting, you need to pay first and last month's rent, plus a $650 security deposit, for a total of:

$665 x 2 + $650 = $1,980

So you would need $1,980 to start renting the apartment.

To know more about rule of thumb , refer here :

https://brainly.com/question/943703#

#SPJ11

The volume of a cylinder is 336m3 . what is the volume of a cone with the same radius and height?

Answers

336 abandoning conductive Chuck nonunion from DM me B in F oh ha you say J

Catherine talks on the phone for 3/4 of an hour every night. How many hours dose she talk on the phone is one week. No decimals pls!

Answers

Answer: 5 1/4

Step-by-step explanation: 3/4 times 7/1 equals 5.25

Turn the decimal to a fraction and get 5 1/4

It is typical for the person who is most difficult to convince in an argument to say that everyone else is stubborn. group of answer choices displacement sublimation projection rationalization regression

Answers

"It is typical for the person who is most difficult to convince in an argument to say that everyone else is stubborn." refers projection (option c).

To understand projection in mathematical terms, we can think of it as a function f(x), where x represents the individual's own thoughts or emotions. The output of the function, f(x), represents the projection of these thoughts or emotions onto someone else.

In the case of arguments, the person who is most difficult to convince may have a high value of x, indicating that they are feeling stubborn. However, instead of accepting this feeling, they project it onto others, resulting in a high value of f(x) for those around them.

Projection is just one example of the many defense mechanisms that humans use to protect themselves from unpleasant thoughts or emotions. Understanding these mechanisms can help us to better navigate difficult situations, including arguments, and improve our communication skills.

Hence the correct option is (c).

To know more about argument here

https://brainly.com/question/31218461

#SPJ4

Select the correct answer from each drop-down menu.
Based on the two triangles shown, what can be concluded?
An angle opposite the longest side of a triangle is the side

Answers

The two triangles shows that an angle opposite the longest side of a triangle is the largest angle

Making conclusions from the two triangles shown

From the question, we have the following parameters that can be used in our computation:

The two triangles

From the triangles we have the largest angles to be

C = 117.3 and E = 93 degrees

The lennths opposite these sides aere

AB = 6 and DF = 11.94

These lengths are the longest segments on their respective triangles

This means that an angle opposite the longest side of a triangle is the largest angle

Read more about triangles at

https://brainly.com/question/14285697

#SP{J1

What is the measure of angle A? Enter your answer as a decimal in the box. Round only your final answer to the nearest hundredth. m∠A=° Right triangle A B C, with right angle C B A. Side A B is three centimeters, side B C is four centimeters, and side C A is five centimeters.

Answers

The measure of angle A is given as follows:

m < A = 53.13º.

What are the trigonometric ratios?

The three trigonometric ratios are the sine, the cosine and the tangent, and they are defined as follows:

Sine of angle = length of opposite side to the angle divided by the length of the hypotenuse.Cosine of angle = length of adjacent side to the angle divided by the length of the hypotenuse.Tangent of angle = length of opposite side to the angle divided by the length of the adjacent side to the angle.

Angle A is opposite to a side of length 4 cm, while the hypotenuse is of 5 cm, hence it's measure is obtained as follows:

sin(A) = 4/5

m < A = arcsin(4/5)

m < A = 53.13º.

Missing Information

We have a right triangle, in which the side length opposite to angle A is BC = 4, while the hypotenuse is CA = 5.

More can be learned about trigonometric ratios at brainly.com/question/24349828

#SPJ1

The preimage and image of WXYZ are shown. Use coordinate notation to describe the translation.

Answers

The image of  WXYZ moves two unit to the right and 5 unit down. The coordinate is (x+2, y-5).

The polar system of coordinates is a coordinate system with two dimensions in which the location of each point can be determined by its distance from the origin, a fixed point, and its angle from the polar axis, also known as the polar axis of rotation. A ray beginning at the origin and extending outward at a set angle, typically 0 degrees or horizontal, is how the polar axis is typically depicted. The image of  WXYZ moves two unit to the right and 5 unit down. The coordinate is (x+2, y-5).

To know more about coordinate, here:

https://brainly.com/question/16634867

#SPJ1

Other Questions
case study, I have a child wellness home visit and I saw mum with two baby, I have noticed baby in dirty nappy, with dirty clothes, their room was in mesh things was lying everywhere and 6 of them staying in one room, they having financial problems to support kids and requesting food parcel district health team. need to 800 words of reflection writing with use of gibbs cycles and also reference list. thanks Who joins bernardo, the watchman, on his watch ofthe castle?francisco and horatioo marcellus and horatioo prince hamlet and marcelluso francisco and prince hamlet Expresa cada proposicin mediante la grfica, conjunto, desigualdad y la notacin de intervalo.a. La estatura de los jugadores de un equipo de baloncesto es menor a 1,98 m y mayor o igual a 1,82 m.b. Los niveles normales de glucosa en ayunas en un ser humano deben ser mayores o iguales a 70 mg/dl y menores que 100 mg/dl.c. El tiempo que tarda una persona en llegar a su trabajo es mayor a 5/6 hora y menor o igual a 3/2 hora.d. La categora juvenil es para personas mayores a 15 aos y menores o iguales a 18 aos.e. La temperatura de la ciudad de Cali regularmente es menor a 31 C y mayor o igual a 19C. Using either a concession or a counterargument in an essay makes an author seem less knowledgeable and inconsistent because they cannot make up their mind which side they are on. TrueFalse(40 points) will give brainiest for effort Complete the story by supplying the adverbs for the 22 blanks that will fit the context of the sentence. Use your imagination to make the finished story an interesting one. Joe 1_____ mopped his brow. Hunting for a Christmas tree had been hard work! He tried to think what Christmas would be like without a 2______ decorated tree. 3_____ he was in another world. He was in the courtyard of an inn. He saw people 4_____ moving about and whispering 5_____ about the unreasonable demands the government had made on them. The night was cool, and everyone was trying to make final preparations for eating and sleeping. Where would they sleep, and how would they feed everyone? In the midst of the hustle and bustle created by all those important decisions, Joe noticed one small donkey 6_____ entering the busy courtyard lead by a 7_____ looking man. There seemed to be an air of watchfulness as the 8_____ weary gentleman lifted a beautiful young lady down from the donkey's back. "Why is everyone staring at them?" Joe wondered. He noticed the lady had an unsteady step and seemed to lean 9_____ upon the arm of her husband. The donkey gazed 10_____, as if watching for someone to lend a helping hand to this special couple. None seemed to care; everyone was dashing about thinking of himself. 11_____ the inn-keeper began to shake his head at the man. There wasn't a bed in his inn! They would have to search elsewhere. Joe moved up closer to the man. He heard the man 12_____ whisper to his wife, "Don't worry, Mary, God knows we need a room. He will provide. "In that moment Joe knew he must help. What could he do? He thought 13_____ and decided to ask the man if he could wait with Mary and the donkey while the man searched for a room. "That is a 14_____ kind deed, young man. If my wife doesn't find a place very soon she may faint, for she is 15_____ tired!" "The inn-keeper has one more stall for the donkey. Maybe you can help by leading the donkey while I help Mary!" 16_____ Joe untied the gentle animal and led it to the back of the inn. T Select the best response to the following questions, Which diversity dimension is the relatively stable set of behaviors, attitudes, and emotions displayed over time that makes people different from each other? Disposition Agreeableness Conscientiousness Personality Select the best response to the following questions, Which term describes the degree to which someone is active, assertive, gregarious, sociable, talkative, and enerozed by others Personality Conscientiousness Emotional stability Extraversion A builder is creating a scale drawing of a plot of land as shown. The original plot of land is 335 meters wide. The drawing uses a scale factor of 1500. Find the missing side length of the original plot of land in meters and the missing side length of the scale drawing in centimeters.Original plot of land's length: mScale drawing width: Two scout patrols start hiking in opposite directions. Each patrol hikes 5 kilometers. Then the scouts turn 90 degrees to their right and hike another 6 kilometers. How many kilometers are there between the two scout patrols? A right square pyramid is shown. A plane intersects the pyramid through the apex and is perpendicular to the base. Calculate the decrease in temperature when 3.00 L at 28.0 C is compressed to 1.00 L. Translate the following balanced chemical equation into words.PCl5(s) + 4H2O(l) H3PO4(aq) + 5HCl(aq)A. Phosphorus pentachloride and water yield phosphoric acid and hydrochloric acid.B. Phosphorus pentachloride and phosphoric acid yield water and hydrochloric acid.C. Phosphorus pentachloride and water yield phosphorous acid and chloric acid.D. Phosphorus hexachloride and water yield phosphoric acid and hydrochloric acid. Which statement about a dilation with a scale factor of 3 is true?O Three is added to each side in the pre-image to find the corresponding side length in the image.O Three is subtracted from each side in the pre-image to find the corresponding side length in the image.Each side in the pre-image is multiplied by three to find the corresponding side length in the image.Each side in the pre-image is divided by three to find the corresponding side length in the image. An airplane is circling an airport at a height of 500m. the angle of depression of the control tower of the aiport is 15 degrees. what is the distance between the airplane and the tower A speech about the effects of cyber bullying on students which statement is true? $9,900 while general, selling, and administrative expenses totaled $3,600. the company produced 7,100 units and sold 4,400 units at a price of $7.10 a unit. what is the amount of finished goods inventory on the balance sheet at year-end? Samantha is working through the Client Needs Assessment process. Which 4 areas should she focus on to gather information for her client?(Select all that apply) Who is the client Why did the client start their business What services does the client need How will the client work be completed Which team member will do the work When does the client need their work completed gradeMathZ.1 Scale drawings of polygons WEALanguage8Polygon P is a scaled copy of polygon N.104Learn with an example4201640Polygon NPolygon PWhat scale factor takes polygon N to polygon P?for10Watch a video 3. What is the largest number that divides 626, 3127 and 15628 and leaves remainders of 1, 2 and 3 respectively? A magic square is shown below. Every row, column and long diagonal adds to the same total. Each number can only be used once. Copy the magic square into your book and complete it using the numbers provided. 2 Numbers to use 3 -1 0 3 X 2 4 Magic square -3 2 1 -4 -2